Multivariable Calculus Limit process

In summary, the limit of the function (x^6-y^6)/(x^3-y^3) as (x,y) approaches (0,0) does not exist. This was determined by approaching along different paths, such as y=0 and x=0, and using L'Hospital's rule. However, approaching along the curve y=x^6 showed that the limit is 0. Additionally, using polar coordinates also results in a limit of 0, proving that the limit does not exist for all paths. The 2-path rule only proves that the limit does not exist, while the polar coordinates method proves both cases.
  • #1
theno1katzman
21
0

Homework Statement



Determine whether the limit exists; if it does, what is it?

Homework Equations



take the limit as (x,y) -> (0,0) of f(x,y) where f(x,y) = (x^6-y^6)/(x^3-y^3)

The Attempt at a Solution



What i started doing was approaching along the line y=0 and that would give
lim as (x,0) -> (0,0) x^6/x^3 simplifying to the lim as (x,0) -> (0,0) x^3 which is 0.

Then I approached along the line x=0 and that would give
lim as (0,y) -> (0,0) -y^6/-y^3 simplifying to the lim as (0,y) -> (0,0) y^3 which is 0.

Then i approached along the line y=x
lim as (x,x) -> (0,0) (x^6-x^6)/(x^3-x^3) = 0/0 which is an indeterminate form so I use L'Hospital's rule a few times and continue to get 0/0. This rational function is the same as saying (x-x)/(x-x).

I conclude that the limit does not exist becuase it can not be evaluated any further. Am i correct? Is there something else I shoudl be doing? This homework problem is due March 22.
Thank you for your help.
 
Physics news on Phys.org
  • #2
Welcome to PF!

Hi theno1katzman! Welcome to PF! :smile:

(try using the X2 tag just above the Reply box :wink:)
theno1katzman said:
take the limit as (x,y) -> (0,0) of f(x,y) where f(x,y) = (x^6-y^6)/(x^3-y^3)

Then i approached along the line y=x
lim as (x,x) -> (0,0) (x^6-x^6)/(x^3-x^3) = 0/0 which is an indeterminate form so I use L'Hospital's rule a few times and continue to get 0/0. This rational function is the same as saying (x-x)/(x-x).

I conclude that the limit does not exist becuase it can not be evaluated any further. Am i correct?

hmm … I've a nasty feeling that the domain of the function isn't supposed to include the line x = y, in which case you can't approach along that line.

Try approaching along a curve on which f(x,y) is constant. :wink:
 
  • #3
Thanks Tim, when I tried approaching (0,0) along the curve y=e^x I will get
limit of (x,e^x)->(0,0) of (x^6-e^(6 x))/(x^3-e^(3 x)) = 1

This is a different answer for the limit above which was 0, therefore the limit of the original function as (x,y) approaches (0,0) does not exist.
 
  • #4
theno1katzman said:
Thanks Tim, when I tried approaching (0,0) along the curve y=e^x …

(please use the X2 tag just above the Reply box :wink:)

erm :redface: … noooo …

y = ex doesn't get anywhere near (0,0), does it? :wink:

Try again, to find a curve on which f(x,y) is constant (you may have to solve a quadratic). :smile:
 
Last edited:
  • #5
Let me know if this is correct. If I approached (0,0) along the path y=x n where n is any positive integer for example y=x2 then after factoring I would get

lim (x,x2)-> (0,0) x3(x2-x+1)/(x2+x+1) = 0/1 = 0.

Therefore the limit exists and is 0.
 
  • #6
I didn't follow that … for y = x2, it would be x3(1 - x6)/(1 - x3), which does --> 0.

But that doesn't prove it for all paths, does it?

Try my previous hint, to find a curve on which f(x,y) is constant.
 
  • #7
[tex]y = x^6 + c [/tex] [tex] lim (x,y) -> (0,0) f(x,y) [/tex]
 
  • #8
Did you notice that the numerator is a difference of squares ?
 
  • #9
Ahh yes I will simplify this to [tex]x^3+y^3[/tex] and work the problem again
 
  • #10
theno1katzman said:
Ahh yes I will simplify this to [tex]x^3+y^3[/tex] and work the problem again

So the limit = ?

The polar coordinates works also :)
 
  • #11
limit would be 0 if I use polar coordinates, question, would polar coordinates prove the limit exists for all paths?
 
  • #12
theno1katzman said:
limit would be 0 if I use polar coordinates, question, would polar coordinates prove the limit exists for all paths?

Yes.
There is a big difference in solving multivariable limits between the 2-path rule (which is y=mx .. etc) and the polar coordinates method.
2-path rule proves only that the limit D.N.E and does not prove the existence of the limit.
Polar coordinates method proves both cases.
 
  • #13
Thank you, I wish my professor would have clarified that in class.
 

1. What is the definition of a limit in multivariable calculus?

In multivariable calculus, a limit is defined as the value that a function approaches as the input values get closer and closer to a specified point, known as the limit point.

2. How is the limit process different in multivariable calculus compared to single variable calculus?

The limit process in multivariable calculus is different because it involves considering the behavior of a function in multiple dimensions, rather than just one. This means that the limit must be approached from different directions and can have different values depending on the path taken to reach it.

3. Can you give an example of a multivariable limit?

Sure, an example of a multivariable limit is the function f(x, y) = (x^2 + y^2)/x, where the limit is taken as (x, y) approaches (0, 0). This limit does not exist because the limit values are different depending on the path taken to approach (0, 0).

4. How do you determine if a multivariable limit exists?

A multivariable limit exists if the function approaches the same value regardless of the path taken to reach the limit point. This can be determined by checking the limit values along different paths or by using the Squeeze Theorem.

5. What are some real-world applications of multivariable limits?

Multivariable limits have many applications in fields such as physics, engineering, and economics. For example, they can be used to analyze the behavior of a system in multiple dimensions, optimize functions with multiple variables, and model complex systems.

Similar threads

  • Calculus and Beyond Homework Help
Replies
10
Views
439
  • Calculus and Beyond Homework Help
Replies
4
Views
560
  • Calculus and Beyond Homework Help
Replies
5
Views
872
  • Calculus and Beyond Homework Help
Replies
1
Views
460
  • Calculus and Beyond Homework Help
Replies
8
Views
1K
  • Calculus and Beyond Homework Help
Replies
6
Views
854
  • Calculus and Beyond Homework Help
Replies
2
Views
542
  • Calculus and Beyond Homework Help
Replies
13
Views
2K
  • Calculus and Beyond Homework Help
Replies
1
Views
255
  • Calculus and Beyond Homework Help
Replies
6
Views
2K
Back
Top